LSAT and Law School Admissions Forum

Get expert LSAT preparation and law school admissions advice from PowerScore Test Preparation.

 Administrator
PowerScore Staff
  • PowerScore Staff
  • Posts: 8918
  • Joined: Feb 02, 2011
|
#47141
Complete Question Explanation
(The complete setup for this game can be found here: lsat/viewtopic.php?t=2909)

The correct answer choice is (B)

As with all complete List questions, simply apply the rules to the answer choices.

Answer choice (A): This answer choice is incorrect because window 2 violates the fourth rule (neither O nor P is used in window 2).

Answer choice (B): This is the correct answer choice.

Answer choice (C): This answer choice is incorrect because it violates the first rule (both windows 1 and 2 contain G and P).

Answer choice (D): This answer choice is incorrect because it violates the third rule (window 2 contains both Y and O).

Answer choice (E): This answer choice is incorrect because it violates the second rule (R is used in only one window).

Get the most out of your LSAT Prep Plus subscription.

Analyze and track your performance with our Testing and Analytics Package.